Sketching a curve to how unique solutions

  • Thread starter Thread starter trap101
  • Start date Start date
  • Tags Tags
    Curve
Click For Summary
The discussion centers on graphing the equation x - e^(1-x) - y^3 = 0 and demonstrating that for each x, there is a unique corresponding y. The equation was rearranged to y = (x - e^(1-x))^(1/3), with y-intercept at (0, -e^(1/3)) and x-intercept at (1, 0). Participants explored the asymptotic behavior of the graph, noting that as x approaches infinity, the function behaves like y = x^(1/3), while as x approaches negative infinity, it approaches y = -e^[(1-3)/3]. The conversation emphasized the importance of understanding basic functions and their behaviors to effectively model and graph more complex equations. Overall, familiarity with foundational functions is crucial for analyzing and sketching complex graphs.
trap101
Messages
339
Reaction score
0
Sketch the graph of the equation x - e^(1-x) - y^3 = 0, Show that for each x there is a unique y satisfying the equation.

Attempt:

So the first thing I did was isolate y in order to put the equation in a form to graph (somewhat). did that and got y = (x-e^(1-x))^1/3.

Got the y-int: (0, -e^1/3), x-int: (1,0) [I didn't get this, this was in soln, how'd they get?]

Now after my attempt to graph it, I look at the solution and they say:

we see that the graph is asymptotic to the curve y= x^1/3 as x-->∞ and asymptotic to the curve y = -e^[(1-3)/3] as x --> -∞

How is there any asymptotic behavior? The expression doesn't indicate any restrictions. This is based on Implicit Function Thm by the way
 
Physics news on Phys.org
trap101 said:
Got the y-int: (0, -e^1/3), x-int: (1,0) [I didn't get this, this was in soln, how'd they get?]
They got it by observation. If you can understand the shape of the graphs y=x and y=-e1-x, then adding those together (because we are trying to solve x-e^{1-x}=0) must give us an x-intercept at a reasonably small value of x that is greater than 0. x=1 is what we're looking for.


trap101 said:
Now after my attempt to graph it, I look at the solution and they say:

we see that the graph is asymptotic to the curve y= x^1/3 as x-->∞ and asymptotic to the curve y = -e^[(1-3)/3] as x --> -∞
Well what happens as x\to\infty for e^{1-x}? And try comparing ex with x as x\to\infty, which value grows faster?
 
Mentallic said:
They got it by observation. If you can understand the shape of the graphs y=x and y=-e1-x, then adding those together (because we are trying to solve x-e^{1-x}=0) must give us an x-intercept at a reasonably small value of x that is greater than 0. x=1 is what we're looking for.





Mentallic said:
Well what happens as x\to\infty for e^{1-x}? And try comparing ex with x as x\to\infty, which value grows faster?

Well for the first part as x\to\infty for e^{1-x} the function will tend to 0, but if I'm studying the behavior as x-->∞ don't I have to take into account the other portion of the expression i.e: x from [x-e^(1-x)]? so the e^(1-x) will go to 0, but that other x goes to ∞. That's why I'm confused I know e^x will go to ∞ faster than x alone but that would force my expression to "break" the asymptote.
 
trap101 said:
Well for the first part as x\to\infty for e^{1-x} the function will tend to 0, but if I'm studying the behavior as x-->∞ don't I have to take into account the other portion of the expression i.e: x from [x-e^(1-x)]? so the e^(1-x) will go to 0, but that other x goes to ∞.
Yes exactly, but the whole point of an asymptote is that one function tends to get closer and closer to another.
In this case, as x\to\infty, e^{1-x}\to 0 therefore x-e^{1-x}\to x, which is why your function is asymptotic to y_1=x^{1/3} as x\to \infty.

trap101 said:
That's why I'm confused I know e^x will go to ∞ faster than x alone but that would force my expression to "break" the asymptote.
I understand what you're thinking, and in fact you're right. Asymptotes are defined as being curves that approach each other. They need to get arbitrarily close to a distance of zero between them.
I believe the point your teacher was trying to make is that if you're looking to sketch that graph, then you can get an idea of what happens as x\to -\infty because it'll look as though the graph is asymptotic to y_2=e^{\frac{1-x}{3}} (but it's not!) because the fractional difference between the values of y=\left(x-e^{1-x}\right)^{1/3} and y2 will become very small for x values <0 (but the difference won't approach zero).
 
Ok, in this instance I see what your talking about, now in general from how you went about explaining things I gather what may arise sometimes is that I have to refer to the graphs of functions I'm familiar with i.e e^x, x^1/3, etc and relate them to my given function to give me an idea of the behavior of the "complex" function to graph? It's really appearing more and more that we have to use previously proven facts in order to "model" these complex looking functions. Am I on the right train of thought?

Thanks by the way.
 
trap101 said:
Ok, in this instance I see what your talking about, now in general from how you went about explaining things I gather what may arise sometimes is that I have to refer to the graphs of functions I'm familiar with i.e e^x, x^1/3, etc and relate them to my given function to give me an idea of the behavior of the "complex" function to graph? It's really appearing more and more that we have to use previously proven facts in order to "model" these complex looking functions. Am I on the right train of thought?

Thanks by the way.

Yep, pretty much. You need to have a firm grasp with the basic functions, such as your polynomials, exponentials, logarithms, trig etc. and then as was shown in this problem, it might not be clearly obvious how you'll be needing to graph such functions, but what you can do is consider intercepts, asymptotes, turning points and anything else you feel is deemed necessary. You may even want to plug in some values to get a better idea of what's going on.

Use all the tools you can.
 
Question: A clock's minute hand has length 4 and its hour hand has length 3. What is the distance between the tips at the moment when it is increasing most rapidly?(Putnam Exam Question) Answer: Making assumption that both the hands moves at constant angular velocities, the answer is ## \sqrt{7} .## But don't you think this assumption is somewhat doubtful and wrong?

Similar threads

  • · Replies 10 ·
Replies
10
Views
2K
Replies
7
Views
2K
Replies
2
Views
1K
Replies
2
Views
1K
  • · Replies 6 ·
Replies
6
Views
2K
  • · Replies 1 ·
Replies
1
Views
2K
  • · Replies 5 ·
Replies
5
Views
1K
  • · Replies 3 ·
Replies
3
Views
2K
Replies
5
Views
2K